5 svar
446 visningar
Kiep767 är nöjd med hjälpen
Kiep767 100
Postad: 2 maj 2018 13:23

Ellära

Hej. Jag undrar hur jag ska beräkna resistans i sådant krets.

Hur ska jag tänka? Svar: R

haraldfreij 1315
Postad: 2 maj 2018 14:00

Ett sätt att resonera är att se att systemet är symmetriskt mellan övre och nedre halvan, så då måste lösningen också vara det. Och då kan det inte gå någon ström på tvären i mitten. Alltså kan du plocka bort den resistansen utan att påverka lösningen. Och då tror jag du vet hur man löser uppgiften.

joculator 5289 – F.d. Moderator
Postad: 2 maj 2018 14:03

I detta fall kommer det inte gå någon ström genom det vertikala motståndet i mitten. Ser du varför?

Så du kan faktiskt bara ta bort det. Då återstår 2R parallelt med 2R. Kommer du vidare?

Kiep767 100
Postad: 2 maj 2018 14:10

aa jag kommer vidare men vf går det ingen ström genom den i mitten? Är det för att då möter den mer resistans?

joculator 5289 – F.d. Moderator
Postad: 2 maj 2018 14:13

Potensialen i tre-vägs-krysset oven vertikala-R är samma som potensialen i den andra tre-vägs-krysset. Alltså ingen ström.

Kiep767 100
Postad: 2 maj 2018 14:15

ok nu förstår jag tack för hjälpen.

Svara Avbryt
Close